K
Khách

Hãy nhập câu hỏi của bạn vào đây, nếu là tài khoản VIP, bạn sẽ được ưu tiên trả lời.

12 tháng 2 2019

Đề là vầy đúng không bạn \(5^{n+3}-2^{n+3}+2^{n+1}-5^{n+2}+2^n\)

\(=\left(5^{n+3}-5^{n+2}\right)-\left(2^{n+3}-2^{n+1}-2^n\right)\)

\(=5^{n+2}\left(5-1\right)-2^n\left(2^3-2-1\right)\)

\(=5^{n+2}.4-2^n\left(8-2-1\right)\)

\(=5^{n+1}.2.2.5-2^{n-1}.2.5\)

\(=5^{n+1}.2.10-2^{n-1}.10\)

do \(5^{n+1}.2.10\)chia hết cho 10 với mọi n \

\(2^{n-1}.10\)chia hết cho 10 với mọi n

suy ra \(5^{n+1}.2.10-2^{n-1}.10\)chia hết cho 10 với mọi n

suy ra \(5^{n+3}-2^{n+3}+2^{n+1}-5^{n+2}+2^n\)chia hết cho 10 với mọi n

17 tháng 8 2018

đặt M là n^3 -9n^2+2n.

TH1 : n có dạng 2k => M chia hết cho 2 (bạn  tự cm)

TH2 ; n có dạng 2k+1 => M = (2k+1)^3-9(2k+1)^2+2n

=8k^3+6k+12k^2+1-9(4k^2+4k+1)+2n = ... => M chia hết cho 2 với mọi n (1)

Xét n có dạng 3k => M chia hết cho 3

Xét n có dạng 3k+1 => n^3+2n=(3k+1)^3+2(3k+1)=27k^3+9k+27k^2+6k+3 chia hết cho 3 mà 9n^2 cũng chia hết cho 3 => M chia hết cho 3

Tương tự bạn xét n =3k+2....

=> M chia hết cho 3 vs mọi n (2)

Từ (1) (2) => M chia hết cho 6

17 tháng 8 2018

còn cách lm khác k bạn?

4 tháng 10 2023

Bước 1: Chứng minh công thức đúng cho n = 1. Khi n = 1, ta có: 1² = 1 = 1 . (1 + 1) . (2 . 1 + 1) / 6 = 1. Vậy công thức đúng cho n = 1.

Bước 2: Giả sử công thức đúng cho n = k, tức là 1² + 2² + ... + k² = k . (k + 1) . (2k + 1) / 6. Ta cần chứng minh công thức đúng cho n = k + 1, tức là 1² + 2² + ... + k² + (k + 1)² = (k + 1) . (k + 1 + 1) . (2(k + 1) + 1) / 6.

Bước 3: Chứng minh công thức đúng cho n = k + 1. Ta có: 1² + 2² + ... + k² + (k + 1)² = (k . (k + 1) . (2k + 1) / 6) + (k + 1)² = (k . (k + 1) . (2k + 1) + 6(k + 1)²) / 6 = (k . (k + 1) . (2k + 1) + 6(k + 1) . (k + 1)) / 6 = (k + 1) . ((k . (2k + 1) + 6(k + 1)) / 6) = (k + 1) . ((2k² + k + 6k + 6) / 6) = (k + 1) . ((2k² + 7k + 6) / 6) = (k + 1) . ((k + 2) . (2k + 3) / 6) = (k + 1) . ((k + 1 + 1) . (2(k + 1) + 1) / 6).

Vậy, công thức đã được chứng minh đúng cho mọi số tự nhiên n khác 0.

16 tháng 8 2016

1) Không có số tự nhiên nào nhỏ hơn 1 chia 5 dư 3

2) + Nếu n lẻ thì n + 5 chẵn => n + 5 chia hết cho 2 =>n.(n + 5) chia hết cho 2

+ Nếu n chẵn thì n chia hết cho 2 => n.(n + 5) chia hết cho 2

=> n.(n + 5) luôn chia hết cho 2

3) A = n2 + n + 1

A = n.(n + 1) + 1

a) Do n.(n + 1) là tích 2 số tự nhiên liên tiếp =>n.(n + 1) chia hết cho 2 mà 1 không chia hết cho 2

=> A không chia hết cho 2

b) Do n.(n + 1) là tích 2 số tự nhiên liên tiếp => n.(n + 1) chỉ có thể tận cùng là 0; 2; 6

=> A = n.(n + 1) + 1 chỉ có thể tận cùng là 1; 3; 7 không chia hết cho 5

BN thử vào câu hỏi tương tự xem có k?

Nếu có thì bn xem nhé!

Nếu k thì xin lỗi đã làm phiền bn

Hội con 🐄 chúc bạn học tốt!!!

18 tháng 7 2017

Ta có : n(2n - 3) - 2n(n + 1)

= 2n2 - 3n - 2n2 - 2n

= 2n2 - 2n2 - 3n - 2n

= -5n 

Mà n nguyên nên -5n chia hết cho 5

18 tháng 7 2017

a, Ta có 

n(2n-3)-2n(n+1)=2n2-3n-2n2-2n

=-5n chia hết cho 5

=> DPCM

b, Ta có (2m-3)(3n-2)-(3m-2)(2n-3)

Lại có  (2m-3)(3n-2)=-(3-2m)(3-2n)=(3-2m)(2n-3)

=> (2m-3)(3n-2)-(3m-2)(2n-3)=(2m-3)(3n-2)-(2m-3)(3-2n)=0

=> (2m-3)(3n-2)-(3m-2)(2n-3)=0

=>(2m-3)(3n-2)-(3m-2)(2n-3) chia hết cho 5 

=> DPCM

27 tháng 2 2020

a, gọi 3 số tự nhiên liên tiếp là a;a+1;a+2 (a thuộc N)

+ xét a chia hết cho 3 (đpcm)

+ xét a chia 3 dư 1 => a = 3k + 1      

=> a +  2 = 3k + 1 + 2 = 3k + 3 = 3(k + 1) chia hết cho 3

+ xét a chia 3 dư 2 => a = 3k + 2

=> a + 1 = 3k + 2 + 1 = 3k + 3 = 3(k + 1) chia hết cho 3

vậy trong 3 số tự nhiên liên tiếp luôn có 1 số chia hết cho 3

b, đề không rõ lắm

27 tháng 2 2020

Ta có: \(17^n;17^n+1;17^n+2\) là 3 số nguyên liên tiếp nên luôn có 1 số chia hết cho 3

\(\Rightarrow17^n\left(17^n+1\right)\left(17^n+2\right)⋮3\)

\(\Rightarrow\left(17^n+1\right)\left(17^n+2\right)⋮3\left(17^n⋮̸3\right)\)

=> A \(⋮3\left(ĐPCM\right)\)

22 tháng 10 2017

a ) Xét các trường hợp :

  n chia hết cho 3

  n chia 3 dư 1

  n chia 3 dư 2

Xét n chia hết cho 3 

=> n^3 - n = 3k^3 - 3k nên luôn chia hết cho 3

Xét n chia 3 dư 1

 => n^3 - n = ( 3k+1 )^3 - ( 3k + 1 ) = 27k + 1 - 3k - 1 = 27k - 3k = ( 27 - 3 )k = 24k = ( 3 . 8 )k nên chia hết cho 3

Xét n chia 3 dư 2

=> n^3 - n = ( 3k + 2 )^3 - ( 3k + 2 ) = 27k + 8 - 3k - 2 = ( 27 - 3 )k + 6 = 24k + 6 = 3 ( 8k + 2 ) nên chia hết cho 3

 Vậy n^3 - n luôn chia hết cho 3 

b ) Tương tự